Newest questions tagged geometry - Physics Stack Exchange most recent 30 from physics.stackexchange.com 2024-08-23T13:31:57Z https://physics.stackexchange.com/feeds/tag?tagnames=geometry&sort=newest&session=251c07c040cfd70bab9b7769cfc46769 https://creativecommons.org/licenses/by-sa/4.0/rdf https://physics.stackexchange.com/q/825466 2 Rate of change of surface area is acting weird Gwen https://physics.stackexchange.com/users/413202 2024-08-23T04:43:11Z 2024-08-23T04:58:29Z <p><em>Preface</em>- I made this situation up. An uniform metal sphere of given mass <span class="math-container">$M$</span>, is being continuously fed more mass at its centre at the rate <span class="math-container">$4 \ \text{kg/s}$</span>. If the density stays constant, find the rate of increase in surface area.</p> <p>How I tried to solve this <span class="math-container">$$\rho V=M\implies \rho \frac{dV}{dt}=\frac{dM}{dt}$$</span> So <span class="math-container">$$4π\rho r^2 \frac{dr}{dt}=4\implies \frac{dr}{dt}=\frac{1}{π\rho r^2}$$</span> From here we can easily derive that <span class="math-container">$\frac{dA}{dt}=\frac{8}{\rho r}$</span> using <span class="math-container">$A=4πr^2$</span></p> <p>But we see that rate of increase in surface area is inversely proportional to that of the radius, so if radius increases, will the surface area increase at a slower rate? <strong>I can't actually visualise this, and so I can't analyse or make a diagram</strong></p> <p>Are my methods wrong?</p> https://physics.stackexchange.com/q/825120 1 Is there a general solution to all spherical triangles as described by Arnold Sommerfeld? Aswan Korula https://physics.stackexchange.com/users/421645 2024-08-20T05:42:20Z 2024-08-20T05:46:10Z <p>Arnold Sommerfeld has demonstrated that it is legal to use spherical trigonometry in solving relative velocity compositions. In this work,</p> <p><a href="https://en.wikisource.org/wiki/Translation:On_the_Composition_of_Velocities_in_the_Theory_of_Relativity" rel="nofollow noreferrer">https://en.wikisource.org/wiki/Translation:On_the_Composition_of_Velocities_in_the_Theory_of_Relativity</a></p> <p>he shows his solution (restricted to two right angled congruent triangles) is equivalent to the Lorentz transform.</p> <p>I am looking for a solution to the general case i.e. all spherical triangles whether right angled or congruent or not.</p> https://physics.stackexchange.com/q/825107 1 What is the line of sight distance across the ocean? [closed] SoulSnatcher https://physics.stackexchange.com/users/422820 2024-08-20T02:18:49Z 2024-08-20T07:01:02Z <p>I just watched an experiment where they had a laser at a height of 50 feet (15 metre) above sea level and were able to see it 23 miles (37 km) away at a receiver which was 20 feet (6 metre) above sea level. According to this chart:</p> <p><a href="http://www.totally-cuckoo.com/distance_visible_to_the_horizon.htm" rel="nofollow noreferrer">http://www.totally-cuckoo.com/distance_visible_to_the_horizon.htm</a></p> <p>It says the distance to the horizon at 50 feet (15 metre) is 9.35 miles (15 km) and the distance at 20 feet (6 metre) is 5.92 miles (9.5 km). That's 15.27 miles (24.6 km) total so how was the laser seen at 23 miles (37 km)? Even with a refraction rate of 8% that's only 1.84 miles (3.0 km) for a total of 17.11 miles (27.5 km). So how was the laser seen at 23 miles (37 km)? What am I missing here?</p> https://physics.stackexchange.com/q/824103 -1 How to (partially) cross-divide 3D vectors? [migrated] David Graham https://physics.stackexchange.com/users/332931 2024-08-09T18:33:20Z 2024-08-09T19:18:04Z <p>A useful technique for vector cross-product:</p> <p>It is often stated that there is no way to 3D cross-divide vectors, which is true, with caveats.</p> <p>If we know that <span class="math-container">$ \vec{A} \times \vec{B} = \vec{C} $</span>, and we know the value of <span class="math-container">$\vec{A}$</span> and <span class="math-container">$\vec{C}$</span>, there are an infinite number of possible <span class="math-container">$\vec{B}$</span> vectors that satisfy <span class="math-container">$ \vec{A} \times \vec{B} = \vec{C} $</span>.</p> <p>The vector <span class="math-container">$\vec{B}$</span> can be always decomposed into a vector <span class="math-container">$\vec{B}_{parr}$</span> parallel to <span class="math-container">$\vec{A}$</span> ,</p> <p>and a vector <span class="math-container">$\vec{B}_{p}$</span> perpendicular to <span class="math-container">$\vec{A}$</span>.</p> <p><span class="math-container">$\vec{B}_{parr}$</span> contributes nothing to the value of <span class="math-container">$\vec{A} \times \vec{B}$</span>.</p> <p><span class="math-container">$$ |\vec{B}_{p}| = |\vec{B}|sin(\theta) $$</span></p> <p><span class="math-container">$$ \vec{A} \times \vec{B} = \vec{A} \times \vec{B}_{p} $$</span></p> <p>All vectors that can be expressed as <span class="math-container">$ \vec{B}_{p} + ( scalar * \vec{A} ) $</span> satisfy <span class="math-container">$ \vec{A} \times \vec{B} = \vec{C}$</span>.</p> <p><span class="math-container">$$ \vec{A} \times ( \vec{B}_{p} + (scalar * \vec{A} ) ) $$</span></p> <p><span class="math-container">$$ = \vec{A} \times \vec{B}_{p} + ( scalar * \vec{A} \times \vec{A} ) $$</span></p> <p><span class="math-container">$$ = \vec{A} \times \vec{B}_{p} $$</span></p> <p><a href="https://i.sstatic.net/pBcsJqKf.png" rel="nofollow noreferrer"><img src="https://i.sstatic.net/pBcsJqKf.png" alt="multiple B vectors satisfy cross product" /></a></p> <p>Given <span class="math-container">$\vec{A}$</span> and <span class="math-container">$\vec{C}$</span>, the full exact solution set for <span class="math-container">$\vec{B}$</span> is <span class="math-container">$\vec{B}_{p} + ( scalar * \vec{A} )$</span> .</p> <p>Given only <span class="math-container">$\vec{A}$</span> and <span class="math-container">$\vec{C}$</span>, we cannot determine which <span class="math-container">$\vec{B}$</span> (from the solution set) generated <span class="math-container">$\vec{C}$</span>, but we can determine the unique <span class="math-container">$\vec{B}_{p}$</span>.</p> <p>We can define &quot;vector (partial) cross-divide&quot;</p> <p><span class="math-container">$$ \vec{C}//\vec{A} \equiv (\vec{C} \times \vec{A})/(\vec{A} \cdot \vec{A}) = (\vec{C} \times \hat{A})/|\vec{A}| = \vec{B}_p$$</span></p> <p>You can see this method produce useful results here:</p> <p>https://<a href="https://www.youtube.com/watch?v=_DkXD-cZLQE" rel="nofollow noreferrer">www.youtube.com/watch?v=_DkXD-cZLQE</a></p> <hr /> <p>derivation: <span class="math-container">$$ |\vec{C}//\vec{A}|= |(\vec{C} \times \vec{A})/(\vec{A} * \vec{A})| $$</span></p> <p><span class="math-container">$$ |\vec{C}| = |\vec{A}||\vec{B}|sin(\theta) = |\vec{A}||\vec{B}_{p}| $$</span></p> <p><span class="math-container">$$ |\vec{C}//\vec{A}|= |(|\vec{B}_{p}||\vec{A}||\vec{A})/(|\vec{A}||\vec{A}|)| $$</span></p> <p><span class="math-container">$$ |\vec{C}//\vec{A}|= |(\vec{B}_{p})| $$</span></p> <p><span class="math-container">$\vec{C}//\vec{A}$</span> and <span class="math-container">$\vec{B}_{p}$</span> have identical length, are both perpendicular to both <span class="math-container">$\vec{C}$</span> and <span class="math-container">$\vec{A}$</span>, and the right hand rule points them in the same direction.</p> <p><span class="math-container">$$ \vec{B}_{p} = \vec{C}//\vec{A} $$</span></p> <p>Conversely, <span class="math-container">$ \vec{A}_{perpendicular} = - \vec{C}//\vec{B} $</span>.</p> <p>Proof by component appears as an answer below.</p> https://physics.stackexchange.com/q/823330 0 If we want to apply the formula for torque $= pE\sin\theta$. It is given in the problem that theta~0° [closed] Soumya https://physics.stackexchange.com/users/416911 2024-08-02T09:07:02Z 2024-08-02T14:06:55Z <p>Why do we take sin theta to be theta only and not as sin0° ie., 0?</p> <p><a href="https://i.sstatic.net/gXiKADIz.jpg" rel="nofollow noreferrer"><img src="https://i.sstatic.net/gXiKADIz.jpg" alt="In the question below the solution is provided by taking sin theta to be equal to theta" /></a></p> https://physics.stackexchange.com/q/822963 1 Position of a particle in the plane [closed] deomanu01 https://physics.stackexchange.com/users/376104 2024-07-29T15:57:47Z 2024-07-29T22:50:47Z <p>I'm here to ask a really stupid question just to be sure of its answer. My professor gave us an exercise where we have to determine the Lagrangian of a system that is formed by a circular ring of mass <span class="math-container">$M$</span> and radius <span class="math-container">$R$</span> which is placed in the <span class="math-container">$(x,z)$</span> plane. In this plane, it rotates around one of his point <span class="math-container">$O$</span> fixed in the origin of the axes. Also, there is a particle <span class="math-container">$P$</span> of mass <span class="math-container">$m$</span> that moves on this ring-like object without friction.</p> <p>The Lagrangian has to be written in terms of the generalized coordinates <span class="math-container">$\theta$</span> and <span class="math-container">$\phi$</span>, where <span class="math-container">$\theta$</span> is the angle formed by the segment that connects <span class="math-container">$O$</span> with the center <span class="math-container">$C$</span> of the ring guide with the <span class="math-container">$z$</span> axes and <span class="math-container">$\phi$</span> is the angle formed by the segment that connects the position of the particle <span class="math-container">$P$</span> with the center of the ring guide <span class="math-container">$C$</span> and the <span class="math-container">$z$</span> axes.</p> <p>I have no problem in understanding how to write the Lagrangian itself, but I'm more confused on how the professor wrote the solution for this problem. They stated that the coordinates of the center <span class="math-container">$C$</span> of the guide can be written as: <span class="math-container">$$ \begin{cases} x= R\sin \theta\\ z=-R \cos\theta \end{cases} $$</span> where I would have written since <span class="math-container">$z = R \sin\left(\frac{\pi}{2}-\theta\right)=R \cos\theta$</span>.</p> <p>EDIT: I'll add an image of the problem so that it is easier to visualize the system. This is not official, it is how I understood it. <a href="https://i.sstatic.net/65glEnpB.png" rel="nofollow noreferrer"><img src="https://i.sstatic.net/65glEnpB.png" alt="enter image description here" /></a> Keep in mind that the ring guide can rotate around the origin <span class="math-container">$O$</span>, so <span class="math-container">$\theta$</span> is not fixed.</p> https://physics.stackexchange.com/q/822244 1 Conception of earth's size based on the width of the observable horizon when standing at sea level and the circumference of the earth Sketcher https://physics.stackexchange.com/users/131065 2024-07-22T14:33:17Z 2024-07-27T13:35:36Z <p>As an aid to conceiving of the size of the earth, using the information that the horizontal (left to right or right to left) width of (not the distance to) the observable horizon when standing at sea level is approximately 3.1 miles (5 kilometers) and the circumference is 24,901 miles (40,075 kilometers), can it be said that there are roughly slightly over 8,000 such horizons that fit into the circumference of the earth? I’m not sure if my colloquial, linear or arc notion of the observable horizon standing at a relatively “linear” beach is as correct a definition as one in which one pans one’s head 360 degrees. Actually, based on the distance to a lighthouse about 3 miles away on one side, the 3.1 mile distance for the observable width of the horizon seems a bit low -- by about half. (I got that 3.1 mile number off the internet). Perhaps there may be some perspective diminishment at play somehow.</p> <p>I note the answer to a previously deleted, related question was:</p> <p>If we are on a beach, and our eyes at 3m above the sea level, the horizon is a circle of about 6km. It seems consistent with our intuition.</p> <p><a href="https://i.sstatic.net/fz8i3p96.png" rel="nofollow noreferrer"><img src="https://i.sstatic.net/fz8i3p96.png" alt="horizon equation" /></a></p> https://physics.stackexchange.com/q/821764 1 Discovery of a Formula for Geostationary Orbit Distance: Seeking Expert Feedback [closed] Luca Blonda https://physics.stackexchange.com/users/414855 2024-07-17T19:45:26Z 2024-07-18T00:10:02Z <p>I’m an amateur enthusiast without a formal academic background in mathematics or science. Recently, I stumbled upon an idea and derived a formula that I believe calculates the distance traveled along the geostationary orbit (GEO) as a function of the pointing angle ((<span class="math-container">$\gamma$</span>)) from a point on Earth’s surface. The formula seems to account for the fact that the observation point is not at the center of the Earth but on its surface.</p> <p>Here’s the formula I came up with in Latex: <span class="math-container">$$ d = 2R \arcsin \left( \frac{\sqrt{(R-r)^2 + \left[ r \sin \left( \gamma + \frac{3\pi}{2} \right) + R \right]^2 - 2(R-r) \left[ r \sin \left( \gamma + \frac{3\pi}{2} \right) + R \right] \cos(\gamma)}}{2R} \right) $$</span> Where:</p> <pre><code>• R is the distance of the GEO from the center of the Earth, • r is the radius of the Earth, • gamma is the pointing angle relative to the vertical direction. </code></pre> <p>I’ve tested the formula for various values of <span class="math-container">$\gamma$</span>, and it seems to work well for the edge cases (<span class="math-container">$\gamma = 0 \text{ and } \gamma = \pi$</span>), and it also shows an exponential-like behavior for angles close to <span class="math-container">$\pi/2$</span>, which makes sense geometrically as the distance to the orbit increases.</p> <p>I’m curious to know what more experienced mathematicians and physicists think about this. Is this formula known in the literature? Does it correctly account for the geometrical complexities involved?</p> <p>Any feedback, corrections, or references to similar known formulas would be greatly appreciated. Thanks in advance for your insights!</p> https://physics.stackexchange.com/q/821057 0 Throwing a ball in the air tensorman666 https://physics.stackexchange.com/users/392115 2024-07-10T14:24:31Z 2024-07-10T15:01:57Z <p>When we throw a ball in the air, we know that if we do not throw it too high, then g can be held constant over the trajectory and we can approximate the curve by a parabola. However we also know that if we project a body at a height from the surface of the earth and the velocity is lower than the first cosmic velocity <span class="math-container">$v&lt;(\frac {GM}{R})^{1/2}$</span> then the path followed by the body will be an ellipse. So then we can say that the trajectory is an ellipse. So in this case we approximated an ellipse by a parabola. Can we always do this?</p> https://physics.stackexchange.com/q/821020 0 Einstein's notion of "covariant" Awe Kumar Jha https://physics.stackexchange.com/users/213213 2024-07-10T06:21:56Z 2024-07-11T16:40:24Z <p>In his <i>The Meaning of Relativity</i>, pg. <span class="math-container">$11-12$</span>, Einstein explains the notion of &quot;covariant&quot; along the following lines: <br/> Consider a point <span class="math-container">$\mathbf x$</span> on a straight line <span class="math-container">$\mathbf x -\mathbf A=\lambda\mathbf B$</span> in <span class="math-container">$\mathbb R^3$</span>. Without a loss of generality we can assume that <span class="math-container">$\mathbf B^T\mathbf B =1$</span>. Now, consider an orthogonal linear transformation (OLT) from the <span class="math-container">$K_{\mathbf x}$</span> frame to <span class="math-container">$K'_{\mathbf x}$</span> defined by: <span class="math-container">$$\mathbf x'=\mathbf a +b\mathbf x$$</span> where <span class="math-container">$b=[b_{\mu\nu}]$</span> is the transformation matrix satisfying <span class="math-container">$b^Tb=I$</span>. Then the straight line in our transformed frame becomes <span class="math-container">$\mathbf{x'-A'=}\lambda\mathbf B'$</span> with <span class="math-container">$\mathbf A'=b^T\mathbf A$</span> and <span class="math-container">$\mathbf B'=b^T\mathbf B$</span>. It therefore implies that straight lines have an underlying property which is independent of the system of coordinates. Formally, this depends upon the fact that the (vector) quantity <span class="math-container">$\vec q :=(\mathbf x -\mathbf A)-\lambda\mathbf B$</span> is transformed as components of an interval, <span class="math-container">$\Delta\mathbf x$</span>. If <span class="math-container">$\vec q=\vec 0$</span> for one system of Cartesian coordinates, then <span class="math-container">$\vec {q'}=\vec 0$</span> for all systems. Thus we can say that the equation of a straight line is &quot;<i>covariant</i>&quot; with respect to OLT's. <br/> <br/> What I think of the assumption <span class="math-container">$\mathbf B^T\mathbf B =1$</span> is that here Einstein is referring to set of <em>homothetic transformations</em> centered at the point <span class="math-container">$\mathbf A,$</span> that an OLT maps a homothety centered at <span class="math-container">$\mathbf A$</span> to one centered at <span class="math-container">$\mathbf A'$</span>, preserving their scaling factor <span class="math-container">$\lambda.$</span> So, straight lines are mapped to straight lines by an OLT, the points on which are therefore called &quot;covariant&quot; vectors. Is my interpretation correct?</p> <h3> Edit </h3> After further reading I find that he is talking about the covariance of laws of physics expressed as equations: a law <span class="math-container">$F=0$</span> where <span class="math-container">$F$</span> may be a vector or scalar or tensor expression in general, if this law preserves its nature under a transformation, that is if it is transformed to <span class="math-container">$F'=0$</span> under the said transformation where <span class="math-container">$F'$</span> has similar mathematical form as <span class="math-container">$F$</span>, then the law is said to be covariant with respect to that transformation. https://physics.stackexchange.com/q/820897 2 Potential of circle and $n$-gon lesobrod https://physics.stackexchange.com/users/180484 2024-07-09T04:54:58Z 2024-07-09T12:00:52Z <p>Consider power central fields <span class="math-container">$f \sim r^k, - \infty &lt; k &lt; \infty$</span>, and unit circle, &quot;charged&quot; in the sense of the field.<br /> We will be interested in the <em>potential within the circle</em>, i.e. scalar sum or integral.<br /> The potential <span class="math-container">$\psi$</span> of a charge or small element will also be a power,<br /> except <span class="math-container">$f \sim r^{-1}$</span>, then <span class="math-container">$\psi \sim \ln (r)$</span></p> <p><strong>Var 1.</strong> The charge is distributed continuously.<br /> It is known that for <span class="math-container">$f \sim r^{-2}, \psi \sim r^{-1}$</span> the force acting on the test charge <em>inside the spheres</em> are <span class="math-container">$0$</span> and the potential is constant. This is proved not only by Gauss’s theorem,<br /> but also through the consideration of sites cut out by opposite bodily angles.<br /> In the case of circle and other powers I find it difficult to apply the integral theorems.</p> <p>Applying the idea of angles and arcs, it turns out that for <span class="math-container">$f \sim r^{-1}, \psi \sim \ln (r)$</span> the full potential inside the circle must be constant, and this is confirmed by the trial by calculations.<br /> Question 1.1 How to prove it?<br /> Question 1.2 Are there other central fields, perhaps not power law,<br /> for which the potential inside the circle is constant?</p> <p><strong>Var 2.</strong> If we replace the circle with a right <span class="math-container">$n$</span>-gon, the angle dependence appears.<br /> But not always! As revealed intuitively, and <a href="https://math.stackexchange.com/questions/4940424/sum-of-powers-of-distances-between-points-in-concentric-circles/4941839#4941839">now proven</a>:</p> <p>If <span class="math-container">$k = 2 d &gt; 0$</span>, the potential does not depend on the angle at <span class="math-container">$d &lt; n$</span></p> <p>Question 2.1 Can the linked reasoning be generalised and supplemented (eg for <span class="math-container">$k&lt;0$</span>)?<br /> Question 2.2 Even in cases of other <span class="math-container">$k&gt;0$</span>, the potential dependence on the angle is very weak.<br /> For example, at <span class="math-container">$k = 1, n = 3$</span> and moving the test charge around a radius of <span class="math-container">$0.5$</span><br /> potential varies within <span class="math-container">$3.14 &lt; \psi &lt; 3.23$</span><br /> And for <span class="math-container">$n = 4$</span> <span class="math-container">$4.23 &lt; \psi &lt; 4.25$</span><br /> No &quot;approximation&quot; of a circle by a triangle and a square can be said. So why this is so?</p> https://physics.stackexchange.com/q/820881 0 Why does moment of inertia stop at 1/2 as solidness of a cylinder increases? Shelby Longbottom https://physics.stackexchange.com/users/362854 2024-07-09T01:14:58Z 2024-07-09T22:43:06Z <p>So I have two things about moment of inertia:</p> <p>The I for a hoop is <span class="math-container">$I=MR^2$</span> and the I for a solid disk is <span class="math-container">$I=\frac{1}{2}MR^2$</span></p> <p>I've noticed that as &quot;solidness&quot; goes up from 0% (completely unsolid, hoop) to 100% (solid disk) the moment of inertia lowers until it's exactly 1/2 of the hoop (when it's solid).</p> <p>Using the equation for a cylinder bound by <span class="math-container">$R_1$</span> (inner radius) and <span class="math-container">$R_2$</span> (outer radius) of <span class="math-container">$$I=\frac{1}{2}M(R_1^2+R_2^2)$$</span> we can think of &quot;solidness&quot; as the inverse percentage <span class="math-container">$R_1$</span> is of <span class="math-container">$R_2$</span>.</p> <p>I plotted this on a graph taking <span class="math-container">$R_1$</span> as the x-axis and moment of inertia I as the y-axis (and assumed <span class="math-container">$R_2=1$</span> and <span class="math-container">$M=1$</span>): <a href="https://i.sstatic.net/0kPlu8GC.jpg" rel="nofollow noreferrer"><img src="https://i.sstatic.net/0kPlu8GC.jpg" alt="enter image description here" /></a></p> <p>The question I have is: why does moment of inertia stop at 1/2 as the disk becomes 100% solid? Is this something inherent to the geometry of circles where <span class="math-container">$I=\frac{1}{2}$</span> if there's a mass of 1 kg distributed equally over a circle of radius 1m? Why is it specifically <span class="math-container">$I=\frac{1}{2}$</span>?</p> <p>Also, why is the graph parabolic from 1/2? I know it's parabolic because it's based on the <span class="math-container">$I=\frac{1}{2}M(R_1^2+R_2^2)$</span> parabolic equation but <em>intuitively</em> why does I go up parabolically?</p> https://physics.stackexchange.com/q/818193 1 Finding Exterior Confining Pressure from Interior Pressure Point for a Solid Disk Modestas Botha https://physics.stackexchange.com/users/406118 2024-06-13T00:16:42Z 2024-06-13T03:24:28Z <p>Essentially, I've been wrapping the pictured object tightly with string to exert a confinement pressure on its exterior. It's been difficult however to make a good estimate of how much pressure is exerted on the object. Theoretical models I've found of string wrapped around a cylinder/disk seem to be too simple for making precise measurements. Right now I'm using a pressure sensor to take measurements from the center of the object where the two interior pillars meet (the sensor doesn't work well with large surface areas). I can make precise measurements of the pressure within the pillars using the sensor, the pressure between the pillars clearly doesn't give the value of the pressure exerted on the exterior, as it doesn't match other setups I've tried or the models.</p> <p>I've tried to work out how the pressure between the interior pillars relates to the confinement pressure. However, I don't have much confidence in my results.</p> <p>Just in case this must be solved numerically, the dimensions are as follows:</p> <p>Radius of the disk: 17.5 mm Radius of the hollow region: 11 mm Width/Height of disk: 14 mm Radius of central pillar contact surface: 4.5 mm Depth of lipped edge: 4.5 mm</p> <p><a href="https://i.sstatic.net/82OPVIdT.png" rel="nofollow noreferrer"><img src="https://i.sstatic.net/82OPVIdT.png" alt="Pictured on the left is the object of discussion from a front-face view. On the right is an image looking vertically upward at the object's cross section." /></a></p> https://physics.stackexchange.com/q/817690 5 Is obeying the parallelogram law of vector addition sufficient to make a physical quantity qualify as a vector? HerrAlvé https://physics.stackexchange.com/users/368779 2024-06-08T17:08:20Z 2024-06-15T08:30:57Z <p>I know that obeying <a href="https://en.wikipedia.org/wiki/Euclidean_vector#Addition_and_subtraction" rel="nofollow noreferrer">the parallelogram rule of vector addition</a> is a necessary condition for vectors. But is it sufficient? In other words, can there be a quantity that is added using the method but does not have either of magnitude or direction?</p> <p>The source of my confusion is that many physics texts list three properties that a physical quantity must have in order to be classified as a vector. It should: have a magnitude, have a direction and be added using the parallelogram rule of vector addition.</p> <p>But I think that the third condition, in and of itself, is sufficient, i.e. the first two fall under its umbrella. So my question basically is -- are the magnitude and direction conditions in the (elementary, <em>physics</em>) definition of vectors redundant, if the obedience of the parallelogram rule has already been taken care of?</p> <p>In my opinion, the answer is obviously yes, but one can never be too sure.</p> https://physics.stackexchange.com/q/817282 25 Why are spherical shapes so common in the universe? Ishaan https://physics.stackexchange.com/users/403880 2024-06-05T14:18:46Z 2024-06-08T12:08:00Z <p>I have a simple question. Why are most objects in the observable universe spherical in shape? Why not conical, cubical, cuboidal for instance? I am furnishing a few points to justify this statement:</p> <ol> <li><p>Most of the planets in the universe are spheroids.</p> </li> <li><p>Other heavenly bodies (e.g.: stars and moons) are also spherical.</p> </li> <li><p>Even elementary particles like atoms are best described as spheres (I admit that they don't have well-defined boundaries, but we still use terms like atomic radius when describing atoms.)</p> </li> </ol> <p>So, my primary question boils down to this single line:</p> <blockquote> <p>Why is the spherical shape so ubiquitous in the observable universe (both in the microscopic and the macroscopic world)?</p> </blockquote> https://physics.stackexchange.com/q/816402 0 Describing force accumulation trend of an infinite volume with evenly distributed radiative sources WhetScience https://physics.stackexchange.com/users/296036 2024-05-30T01:38:38Z 2024-05-30T01:38:38Z <p>I am looking for confirmation if I've built my equation properly.</p> <p>My goal is to describe the change in force over time at a given point if evenly distributed radiators (in-phase or cumulative energy/force) in an effectively infinite volume begin emitting simultaneously. One might visualize this as a matrix of light bulbs, but I am strictly calculating for non-interfering radiation (no phase considerations).</p> <p>Sir Isaac Newton’s inverse square law (<span class="math-container">$F=\frac{1}{r^2}$</span>) describes the reduction of power from a given source over distance. But I am interested in the influence of multiple sources on a point in the center, so I'm looking at the geometry of progressive spheres with equal surface density of sources (<span class="math-container">$A=4r^2$</span>).</p> <p>Attempting to integrate the trend and apply it to gravitational force (the radiative force of interest, 'though any non-interfering radiation should work), my result is the following equation. NOTE: I have not fully simplified it intentionally to show the components used:</p> <p><span class="math-container">$F_{total}=\int_{r_{min}}^{r_{max}}G\frac{M(M4\pi r^2)}{r^2}dr$</span></p> <p>I expect the change in the difference in the range of <span class="math-container">$r$</span> has a direct linear relationship to <span class="math-container">$F_{total}$</span>. This should be the case for any set of <span class="math-container">$r_{min}$</span> and <span class="math-container">$r_{max}$</span> values. The trend I hope to model is, if all radiative sources simultaneously begin emitting, <span class="math-container">$r_{min}=0$</span> and <span class="math-container">$r_{max}=tc$</span> for a given period of time.</p> <p>Again, I'm looking for constructive criticism of my approach and any advice on how to correct or further enhance this equation. Thank you.</p> https://physics.stackexchange.com/q/815847 5 Modified Special Geometry of SUSY Moduli Space TwoStones https://physics.stackexchange.com/users/234583 2024-05-25T10:19:40Z 2024-05-27T10:26:04Z <p>It is known that the Coulomb branches of 5d <span class="math-container">$\mathcal{N}=1$</span> and 4d <span class="math-container">$\mathcal{N}=2$</span> SUSY (both have eight supercharges) satisfy special geometry. This means that there exists a holomorphic prepotential <span class="math-container">$\mathcal{F}$</span> such that the Coulomb branch is given by the hypersurface <span class="math-container">$$\mathcal{CB}=\{T^I,I=0,...,n_V\vert\mathcal{F}[T]=1\},$$</span> where <span class="math-container">$n_V$</span> denotes the number of vector multiplets.</p> <p>It is also known that higher-derivative corrections to supergravities change this special geometry constraint, see e.g. <a href="https://physics.stackexchange.com/questions/812551/prepotential-in-5d-mathcaln-1-with-higher-derivative-correction/812669#812669">this post</a>.</p> <p><span class="math-container">$\textbf{Question:}$</span> How can this new, modified constraint be interpreted? Is the Coulomb branch still given by the hypersurface <span class="math-container">$\mathcal{F}[T]=1$</span> or by the new constraint <span class="math-container">$\mathcal{F}[T]=1+\mathcal{G}[T]$</span>? If the latter is true, does this mean that higher-derivative corrections break special geometry and also the direct product structure of the SUSY moduli space?</p> <p><span class="math-container">$\textbf{Remark:}$</span> From geometric engineering (M-Theory on elliptically fibered CY<span class="math-container">$_3$</span>), I would expect the former to be true. However, I am very unsure about this!</p> <p><span class="math-container">$\textbf{Edit:}$</span> Maybe I should be more explicit on why this correction concerns me. Consider M-Theory on a Calabi-Yau threefold <span class="math-container">$Y$</span>. Then the Kähler moduli space is identified with the <span class="math-container">$\mathcal{N}=1$</span> Coulomb branch in five dimensions. The prepotential <span class="math-container">$\mathcal{F}$</span> is in this case simply the total volume <span class="math-container">$\mathcal{V}_Y$</span> of <span class="math-container">$Y$</span> which sits in a 5d hypermultiplet. Keeping <span class="math-container">$\mathcal{F}\equiv\mathcal{V}_Y=1$</span> fixed gives a constraint on the scalars of the vector multiplets (corresponding to the Kähler parameters of <span class="math-container">$Y$</span>) and this constrained hypersurface is then the Coulomb branch. So far, so good. What I find worrying is that with the inclusion of <span class="math-container">$$S_{ARR}\sim\int_{\mathcal{M}_5}C_IA^I\wedge\mathrm{tr}R^2,$$</span> the condition <span class="math-container">$\mathcal{F}[T]=1$</span> is replaced by <span class="math-container">$\mathcal{F}[T]=1+\mathcal{G}[T]$</span> and it seems to me that now there is no clear distinction between Coulomb and Higgs branch possible anymore. This seems particularly strange as I always considered the direct product structure to be a very rigid condition coming from SUSY and seeing it broken due to a higher-derivative correction feels weird.</p> https://physics.stackexchange.com/q/815049 0 Is displacement vector always the shortest path? Manish https://physics.stackexchange.com/users/391426 2024-05-19T07:56:41Z 2024-05-19T08:05:52Z <p>I read that the <a href="https://en.wikipedia.org/wiki/Displacement_(geometry)" rel="nofollow noreferrer">displacement</a> vector of a particle is the shortest path between its initial and final positions since it's a straight line joining the two points, this holds true for me till a 2D plane but when we enter into 3D figures like a cube then it doesn't seem to always hold true there. So, what's the correct explanation?</p> https://physics.stackexchange.com/q/813254 8 Why are material properties often described by symmetric tensors? Kotlopou https://physics.stackexchange.com/users/226688 2024-05-06T11:56:11Z 2024-05-07T05:56:15Z <p>It seems to me that whenever there is some material parameter for a continuum, it is described by a symmetric tensor. This is the case for the mechanical stress tensor, permittivity of crystals, the inertia tensor, even higher-order nonlinear susceptibility represented by third- and higher order tensors.</p> <p>On the other hand, I don't know any material properties described by general, nonsymmetric tensors.</p> <p>Is there an underlying reason for this that is shared by the examples provided?</p> https://physics.stackexchange.com/q/813148 19 Does the top of a wheel really move at twice the velocity of the center? Manukrishnan P https://physics.stackexchange.com/users/401905 2024-05-05T15:32:00Z 2024-05-10T19:15:39Z <p>According to the physics of a wheel rolling without slipping, the topmost point moves twice as fast as the wheel.</p> <p>But I tried an experiment:</p> <p>Take a wheel on a table and hold a ruler horizontally in contact with the top of the wheel. Roll the wheel without slipping by pushing the ruler forward, such that the center of the wheel moves forward by <span class="math-container">$10 \,\text{cm}$</span>.</p> <p>But the ruler was supposed to move <span class="math-container">$20 \,\text{cm}$</span>, isn't it? But it didn't, it just seemed to move <span class="math-container">$10 \,\text{cm}$</span>. Am I interpreting something wrong?</p> https://physics.stackexchange.com/q/813019 0 Doubt in pitch of screw gauge [closed] Dhyaneshwar https://physics.stackexchange.com/users/385261 2024-05-04T12:39:37Z 2024-05-05T04:24:23Z <p>I have just started learning about screw gauge and I came across this statement about pitch</p> <blockquote> <p>The pitch is the distance between two consecutive threads of a screw which is equal to the distance moved by the screw due to one complete rotation of the cap.</p> </blockquote> <p>I don't understand why it is so. Please explain in simple language.</p> https://physics.stackexchange.com/q/811973 -1 Geometry of spacetime at different length scales [closed] tony schofield https://physics.stackexchange.com/users/401120 2024-04-26T15:21:55Z 2024-04-26T20:19:45Z <p>Does spherical geometry govern physics at the quantum scale?</p> <p>My motivation for this question came from studying non-Euclidean geometry. When we go down from general relativistic length scales to everyday length scales, geometry changes from non-Euclidean to Euclidean. Does this process continue as we move further down to quantum length scales? Does the shape of space continue to change? From a saddle to a flat sheet to a sphere, so to speak.</p> <p>An example would be the 3 parallel lines that meet at the edge of the universe forming a triangle with sum of angles zero. As we shrink this triangle down the angle sum increases until at our scale it is a classical triangle. As this triangle shrinks down to a point does it's angle sum increase? Does it appear convex from our viewpoint and might this explain apparent faster than light travel from our viewpoint?</p> https://physics.stackexchange.com/q/811926 21 Is there such a thing as a "physical" fractal? starseed_trooper https://physics.stackexchange.com/users/150685 2024-04-26T06:50:57Z 2024-04-30T11:18:47Z <p>The recent discovery of a <a href="https://www.mpg.de/21811459/0410-terr-discovery-of-the-first-fractal-molecule-in-nature-153410-x" rel="nofollow noreferrer">molecule that mimics the Sierpinski gasket</a> has spurred headlines identifying it as the first fractal scientists have found in nature. I find these claims highly dubious because it's either entirely impossible for a real fractal to be realized in nature or tons of other structures are trivially fractals even though we would normally not think about them as such.</p> <p>The point of contention here is that, strictly speaking, a fractal must exhibit self-similarity over an <em>infinite</em> number of generations. No such structures are available outside of mathematical idealizations. We'd be tempted to say, then, that this molecule is <em>not</em> a fractal because it lacks the above property.</p> <p>That being said, physicists are comfortable with calling space-time an infinitely differentiable manifold even though infinitely differentiable things are impossibilities. In this sense, we might shove infinities under the rug as physicists and say that the molecule is a fractal because it looks like the Sierpinski gasket over a finite number of generations. This would make it a <em>prefractal</em>, however. Moreover, if we're this lax about self-similarity, it follows that clouds, coastlines, and lightning must be fractals as well, in which case the Sierpinski molecule is obviously not the first fractal in nature</p> <p>So, are there such things as physical fractals or not?</p> https://physics.stackexchange.com/q/811523 5 How do parallel reflected rays meet to form image at infinity? If they never meet then how is image formed? Shivam Gogia https://physics.stackexchange.com/users/400843 2024-04-23T12:19:13Z 2024-04-25T07:23:41Z <p>In my textbooks it is written that when an object is kept at focus, its image is formed at infinity and is real. But how is this possible because parallel lines never meet and it is necessary for rays to meet to form image.</p> <p>Furthermore on youtube there are videos where the object is kept at focus of concave/convex mirror/lens respectively its image is formed very far. How is this posible as parallel rays never meet??</p> <p>Please give me an answer in the context of geometric optics.</p> https://physics.stackexchange.com/q/810172 0 Average Speed in one half of an elliptical orbit Echelon96 https://physics.stackexchange.com/users/399969 2024-04-13T15:37:22Z 2024-04-13T20:40:31Z <p>I was wondering whether the average speed along one half of an elliptical orbit (say in a star planet system) had a closed form exact solution using Kepler's laws.</p> <p>My approach was using the approximation for circumference of the ellipse, dividing by 2 and then dividing the entire thing by half the time period of an elliptical orbit. While this approach is straightforward, it only has an approximate answer.</p> https://physics.stackexchange.com/q/810147 1 How can I see Orion's Belt in winter and summer? Danny Rosenberg https://physics.stackexchange.com/users/399948 2024-04-13T10:07:14Z 2024-04-13T16:38:44Z <p>How can the 23 degree tilt of the Earth enable someone in Argentina to see the same constellation (Orion's Belt) in winter as someone in Britain in summer?</p> https://physics.stackexchange.com/q/809883 13 How do we measure the position of a body? [duplicate] Armadilo https://physics.stackexchange.com/users/399343 2024-04-11T14:20:40Z 2024-04-12T09:52:55Z <p><img src="https://i.sstatic.net/43cxgZLj.png" alt="1" /></p> <p>I am a high school student and I was studying kinematics about position of a body. So, one thing I do not understand in this diagram is the position of woman. Initally, at <span class="math-container">$t=0$</span>, this woman was at <span class="math-container">$x=1.5\,\text{m}$</span> from reference frame; but her hands, her legs or any point of the woman are at different positions. So how we can say that this woman is at <span class="math-container">$x=1.5\,\text{m}$</span>? How we can define the position of the woman?</p> https://physics.stackexchange.com/q/808366 0 Collection efficiency of mirror Sneaxx https://physics.stackexchange.com/users/398750 2024-03-30T18:12:47Z 2024-03-30T18:16:17Z <p>I want to compare plane and parabolic mirror for collection of photon from a point source. Intuitively, parabolic mirror focuses all rays from point source and gives parallel rays therefore has a higher collection efficiency. How do I mathematically prove it? Can someone help in estimating solid angle for both the cases? The mirrors have a height of 1.7mm and width of 4.3mm. The sample is placed around 1.05mm from mirror surface. And focal length in case of parabolic mirror is 0.75mm. How to calculate the efficiency analytically.</p> https://physics.stackexchange.com/q/807751 4 Does the term $d ( \omega_{ab} \wedge \theta^a \wedge \theta^b )$ have any significance? user1379857 https://physics.stackexchange.com/users/157704 2024-03-26T03:19:35Z 2024-03-28T04:56:15Z <p>If <span class="math-container">$\omega_{ab}$</span> is the spin connection 1-form, and <span class="math-container">$\theta^a$</span> are the tetrad 1-forms, then one has the equality</p> <p><span class="math-container">\begin{equation} \int \, d ( \epsilon_{abcd} \omega^{ab} \wedge \theta^c \wedge \theta^d ) = \int d^3 x \sqrt{h} K \end{equation}</span></p> <p>where <span class="math-container">$K$</span> is the extrinsic curvature scalar, and the second term is the GHY boundary term.</p> <p>Is there a similar geometrical interpretation for the term</p> <p><span class="math-container">\begin{equation} \int d ( \omega_{ab} \wedge \theta^a \wedge \theta^b)? \end{equation}</span></p> <p>Does this term have a name, or appear in any known theories? Is it zero?</p> <p>One might also see similarities with these equations involving the Riemann curvature 2-form <span class="math-container">$R_{ab}$</span> <span class="math-container">\begin{equation} \epsilon_{abcd} R^{ab} \wedge \theta^c \wedge \theta^d = R \end{equation}</span></p> <p><span class="math-container">\begin{equation} R_{ab} \wedge \theta^a \wedge \theta^b = R_{\mu \nu \rho \sigma} \epsilon^{\mu \nu \rho \sigma} = 0. \end{equation}</span></p> <p>The first term is the usual Einstein-Hilbert lagrangian whereas the second is the Holst term. Therefore, the term I am asking about is the analog of the Holst term for the GHY boundary term.</p> <p><strong>Edit:</strong> @Adversing has claimed in the comments that the term is zero in Einstein gravity. Does anyone have a proof of this fact?</p> https://physics.stackexchange.com/q/807344 2 Why is the refractive index for light rays travelling in circular paths proportional to $1/r$? davidaddisonsenjaya https://physics.stackexchange.com/users/393292 2024-03-22T19:11:51Z 2024-03-22T20:12:26Z <p>While studying optics, I came across a problem with solution in which the trajectory of light rays was known—circular paths around a fixed point in space, and the question was that of determining the refraction index as a function of the distance <span class="math-container">$r$</span> from that fixed point (given that the refractive index at <span class="math-container">$r = r_0$</span> is <span class="math-container">$n_0$</span>. A bit of a baffling question to me, as without at least knowing where the boundaries are Snell's law seemed to be utterly useless. The only thing that made sense to me was that the refractive index would depend solely on <span class="math-container">$r$</span>, and the light rays were staying in the path with the least change in refractive index. Anyway the solution reads <br> <span class="math-container">$$n \cdot 2 \pi r = constant$$</span> <br> <span class="math-container">$$n = {constant\over r}$$</span> <br> <span class="math-container">$$n = {n_0r_0\over r}$$</span> <br> At first, my intuition justified it as being similar to angular momentum (I understood it as light behaving as a particle), in that <span class="math-container">$v$</span> would also be proportional to <span class="math-container">$1/r$</span>. But then eventually I realized <span class="math-container">$n$</span> is <span class="math-container">$c/v$</span>, so in fact here the speed of light is proportional to <span class="math-container">$r$</span>! The only thread I have at this point is Fermat's principle, but when the trajectory is known and not the refraction index I might as well be reconstructing a chess game by only seeing the final position. Can anyone make sense of this? (I don't suppose there is a nice &quot;formula&quot; for the refraction index function, as in like the one for the Lagrangian in mechanics; <span class="math-container">${1\over2} mv^2 - V(x)$</span>, is there?)</p> -